Difference between revisions of "1989 AHSME Problems/Problem 28"

m (Second Solution)
 
(14 intermediate revisions by 5 users not shown)
Line 1: Line 1:
 +
== Problem ==
 +
 
Find the sum of the roots of <math>\tan^2x-9\tan x+1=0</math> that are between <math>x=0</math> and <math>x=2\pi</math> radians.
 
Find the sum of the roots of <math>\tan^2x-9\tan x+1=0</math> that are between <math>x=0</math> and <math>x=2\pi</math> radians.
  
 
<math> \mathrm{(A)  \frac{\pi}{2} } \qquad \mathrm{(B) \pi } \qquad \mathrm{(C) \frac{3\pi}{2} } \qquad \mathrm{(D) 3\pi } \qquad \mathrm{(E) 4\pi }  </math>
 
<math> \mathrm{(A)  \frac{\pi}{2} } \qquad \mathrm{(B) \pi } \qquad \mathrm{(C) \frac{3\pi}{2} } \qquad \mathrm{(D) 3\pi } \qquad \mathrm{(E) 4\pi }  </math>
 +
 +
== Solution ==
 +
The roots of <math>t^2-9t+1=0</math> are positive and distinct, so by considering the graph of <math>y=\tan x</math>, the smallest two roots of the original equation <math>x_1,\ x_2</math> are between <math>0</math> and <math>\tfrac\pi{2}</math>, and the two other roots are <math>\pi+x_1, \pi+x_2</math>.
 +
 +
Then, from the quadratic equation, we discover that the product <math>\tan x_1\tan x_2=1</math>, which implies that <math>\tan(x_1+x_2)</math> does not exist. The bounds then imply that <math>x_1+x_2=\tfrac\pi{2}</math>. Thus <math>x_1+x_2+\pi+x_1+\pi+x_2=3\pi</math> which is <math>\rm{(D)}</math>.
 +
 +
==Solution 2==
 +
<math>t^2-9t+1=0</math>:
 +
We treat <math>\tan(x_1)</math> and <math>\tan(x_2)</math> as the roots of our equation.
 +
Because <math>\tan(x_1) \times \tan(x_2) = 1</math> by Vieta's formula, <math>x_1 + x_2 = 0.5\pi</math>.
 +
Because the principal values of <math>x_1</math> and <math>x_2</math> are acute and our range for <math>x</math> is <math>[0,2\pi]</math>,
 +
we have four values of <math>x</math> that satisfy the quadratic:
 +
<math>x_1, x_2, x_1+\pi, x_2+\pi.</math>
 +
Summing these, we obtain <math>2(x_1+x_2) + 2\pi</math>.
 +
Using the fact that <math>x_1+x_2=0.5\pi</math>,
 +
we get <math>2(0.5\pi) + 2\pi = 3\pi.</math>
 +
 +
== See also ==
 +
{{AHSME box|year=1989|num-b=27|num-a=29}} 
 +
 +
[[Category: Intermediate Algebra Problems]]
 
{{MAA Notice}}
 
{{MAA Notice}}

Latest revision as of 21:14, 2 March 2019

Problem

Find the sum of the roots of $\tan^2x-9\tan x+1=0$ that are between $x=0$ and $x=2\pi$ radians.

$\mathrm{(A)  \frac{\pi}{2} } \qquad \mathrm{(B) \pi } \qquad \mathrm{(C) \frac{3\pi}{2} } \qquad \mathrm{(D) 3\pi } \qquad \mathrm{(E) 4\pi }$

Solution

The roots of $t^2-9t+1=0$ are positive and distinct, so by considering the graph of $y=\tan x$, the smallest two roots of the original equation $x_1,\ x_2$ are between $0$ and $\tfrac\pi{2}$, and the two other roots are $\pi+x_1, \pi+x_2$.

Then, from the quadratic equation, we discover that the product $\tan x_1\tan x_2=1$, which implies that $\tan(x_1+x_2)$ does not exist. The bounds then imply that $x_1+x_2=\tfrac\pi{2}$. Thus $x_1+x_2+\pi+x_1+\pi+x_2=3\pi$ which is $\rm{(D)}$.

Solution 2

$t^2-9t+1=0$: We treat $\tan(x_1)$ and $\tan(x_2)$ as the roots of our equation. Because $\tan(x_1) \times \tan(x_2) = 1$ by Vieta's formula, $x_1 + x_2 = 0.5\pi$. Because the principal values of $x_1$ and $x_2$ are acute and our range for $x$ is $[0,2\pi]$, we have four values of $x$ that satisfy the quadratic: $x_1, x_2, x_1+\pi, x_2+\pi.$ Summing these, we obtain $2(x_1+x_2) + 2\pi$. Using the fact that $x_1+x_2=0.5\pi$, we get $2(0.5\pi) + 2\pi = 3\pi.$

See also

1989 AHSME (ProblemsAnswer KeyResources)
Preceded by
Problem 27
Followed by
Problem 29
1 2 3 4 5 6 7 8 9 10 11 12 13 14 15 16 17 18 19 20 21 22 23 24 25 26 27 28 29 30
All AHSME Problems and Solutions

The problems on this page are copyrighted by the Mathematical Association of America's American Mathematics Competitions. AMC logo.png